4
$\begingroup$

In a classical noisy regression setting, let $\big(f(x)\big)_{x\in\cal X}$ be a centered Gaussian process of covariance $k$ on a compact $\cal X$, and $\mathcal{F}_n$ be the filtration generated by some noisy measurements $\big(y_i = f(x_i)+\epsilon_i\big)_{i\leq n}$ where $\epsilon_i \overset{iid}{\sim} \mathcal{N}(0,\eta^2)$. Then, let $\mu_n(x) = \mathbb{E}[f(x) \mid \mathcal{F_n}]$ be the posterior mean of the GP given the noisy observations. Even if $f$ is not in the corresponding RKHS $\cal H_k$ with probability 1 in the general case, we know that $\mu_n$ does. Is it possible to prove tail bound for the distribution of $\lVert \mu_n \rVert_{\cal H_k}$ ?

We know that $\lVert \mu_n \rVert_{\cal H_k}^2=\rm Y^T(\rm K + \eta^2 \rm I)^{-1 T}(\rm K+\eta^2\rm I)^{-1}\rm Y$, where $\rm Y$ is the Gaussian vector of the observations $y_i$ and $\rm K$ is the kernel matrix of the $x_i$, but the inverse terms make the upper bound difficult to grasp...

We further know that $\mu_n$ solves the following regularized regression problem: $$\arg\!\min_{\hat{f} \in \cal H_k} \frac 1 2 \lVert \hat{f} \rVert_{\cal H_k}^2 + \frac 1 {2\eta^2} \sum_{i=1}^n \big(y_i - \hat{f}(x_i)\big)^2.$$

$\endgroup$
4
  • $\begingroup$ What do you mean exactly with "upper bound the distribution" of a random variable ? $\endgroup$ Sep 17, 2015 at 13:09
  • $\begingroup$ Oops, I edited the question, I wanted to get high probabilistic results such as $P[\lVert \mu_n \rVert_{\cal H_k} < ...] > 1-\delta$. $\endgroup$
    – Emile
    Sep 17, 2015 at 13:19
  • $\begingroup$ Are the $x_i$ just a fixed sequence of points in $\mathcal{X}$, or something else? Also, what is $\mathcal{H}_k$ meant to denote? $\endgroup$ Sep 17, 2015 at 14:00
  • $\begingroup$ The $x_i$ are an arbitrary sequence of points in $\cal X$, it would be great to prove upper bounds in the worst case. And $\cal H_k$ is the RKHS of kernel $k$. You can find an introduction of the link between RKHS and GP in the chapter 6 of the Rasmussen and Williams' book (available online). $\endgroup$
    – Emile
    Sep 17, 2015 at 14:21

2 Answers 2

2
$\begingroup$

Take the easiest example first: $f$ is brownian, $\eta=0$ (no noise) and $x_i=i/n$. Then $\mu_n$ is piecewise linear and $\int_0^1 \mu'_n(x)^2\ dx$ is a sum of $n$ squared unit variance independent Gaussian variables, i.e. a "chi-square with $n$ degrees of freedom". Everything is explicit in this case, I hope it helps approach more general situations.

$\endgroup$
0
$\begingroup$

It is possible to get tail inequalities of such a quadratic form of a Gaussian vector using the results from (Hsu et al, 2012).

They prove that if $\rm C=A^\top A$ is a psd matrix and $\rm Y$ is a $\sigma$-sub-Gaussian random vector, that is $\exists \sigma \in \mathbb{R},~ \forall \rm \alpha \in \mathbb{R}^n$: $$\mathbb{E}\Big[e^{\rm \alpha^\top Y}\Big] < e^\frac{\lVert \alpha \rVert^2 \sigma^2}{2}~,$$ then for all $t>0$, $$\mathbb{P}\Bigg[\mathrm{Y^\top C Y} = \lVert \mathrm{A Y} \rVert^2 > \sigma^2\Big(\mathrm{tr(C)}+2\sqrt{\mathrm{tr(C^2)}t}+2\lVert\mathrm{C}\rVert t\Big) \Bigg] \leq e^{-t}~.$$

Here the condition is satisfied for $\rm C = (K+\eta^2 I)^{-1}$ which is psd and $\rm Y$ is $\sqrt{2}\lVert A \rVert$-sub-Gaussian.

It is indeed a generalization of chi-square variables as Jean Duchon mentioned.


Hsu, Kakade, Zhang, A tail inequality for quadratic forms of subgaussian random vectors, ECP (2012)

$\endgroup$

Your Answer

By clicking “Post Your Answer”, you agree to our terms of service and acknowledge you have read our privacy policy.

Not the answer you're looking for? Browse other questions tagged or ask your own question.